What is the Inverse Laplace Transform for V(s)=\frac{2s}{(s^{2}+4)^{2}}?

Click For Summary
The discussion focuses on finding the inverse Laplace transform of V(s) = 2s/(s² + 4)². The correct solution is v(t) = (1/2)tsin(2t)u(t), where u(t) is the unit step function. Participants discuss the decomposition of V(s) into partial fractions, noting the need for appropriate coefficients A and B. There is confusion regarding the handling of imaginary terms, but the consensus is that they do not affect the final result. The thread emphasizes the importance of correctly applying the inverse transform techniques to arrive at the solution.
photonsquared
Messages
15
Reaction score
0
1. Find v(t) if V(s)=\frac{2s}{(s^{2}+4)^{2}}

Ans: v(t)=\frac{1}{2}tsin2tu(t)

2. Homework Equations :

V(s)=\frac{a_{n}}{(s-p)^{n}}+\frac{a_{n-1}}{(s-p)^{n-1}}+\cdots+\frac{a_{1}}{(s-p)}
a_{n-k}=\frac{1}{k!}\frac{d^{k}}{ds^{k}}[(s-p)^{n}V(s)]_{s=p}

3. Attempt at a solution:

V(s)=\frac{2s}{(s^{2}+4)^{2}}

V(s)=\frac{2s}{(s^{2}+4)^{2}}=\frac{A}{(s^{2}+4)^{2}}+\frac{B}{(s^{2}+4)}

A=\left[2s-B(s^{2}+4)\right]_{s=2i}

A=4i

B=\frac{d}{ds}\left[2s-B(s^{2}+4)\right]_{s=2i}

B=2

V(s)=\frac{4i}{(s^{2}+4)^{2}}+\frac{2}{(s^{2}+4)}

I am not sure what to do with the imaginary term, but it does not translate to 1/2t, which is what is required for the answer.

?+sin2tu(t)






 
Physics news on Phys.org
photonsquared said:
1. Find v(t) if V(s)=\frac{2s}{(s^{2}+4)^{2}}

Ans: v(t)=\frac{1}{2}tsin2tu(t)

2. Homework Equations :

V(s)=\frac{a_{n}}{(s-p)^{n}}+\frac{a_{n-1}}{(s-p)^{n-1}}+\cdots+\frac{a_{1}}{(s-p)}
a_{n-k}=\frac{1}{k!}\frac{d^{k}}{ds^{k}}[(s-p)^{n}V(s)]_{s=p}

3. Attempt at a solution:

V(s)=\frac{2s}{(s^{2}+4)^{2}}

V(s)=\frac{2s}{(s^{2}+4)^{2}}=\frac{A}{(s^{2}+4)^{2}}+\frac{B}{(s^{2}+4)}
No. Since the denominator, s^2+ 4 is quadratic you need
\frac{2s}{(s^2+4)^2}= \frac{Ax+ B}{(x^2+4)^2}+ \frac{Cx+ D}{x^2+4}
 
Thanks, I'll attempt again.
 
Question: A clock's minute hand has length 4 and its hour hand has length 3. What is the distance between the tips at the moment when it is increasing most rapidly?(Putnam Exam Question) Answer: Making assumption that both the hands moves at constant angular velocities, the answer is ## \sqrt{7} .## But don't you think this assumption is somewhat doubtful and wrong?

Similar threads

  • · Replies 1 ·
Replies
1
Views
1K
  • · Replies 7 ·
Replies
7
Views
2K
  • · Replies 8 ·
Replies
8
Views
1K
  • · Replies 1 ·
Replies
1
Views
1K
Replies
3
Views
2K
  • · Replies 10 ·
Replies
10
Views
2K
Replies
9
Views
2K
  • · Replies 4 ·
Replies
4
Views
3K
  • · Replies 2 ·
Replies
2
Views
1K
  • · Replies 8 ·
Replies
8
Views
2K